AMC MCQ practice questions
AMC MCQ CAT free trial gives you an advantage to prepare for your AMC Medical MCQ Exams. Trial questions brings an overview for AMC Question Bank.
Once you are done with free AMC MCQ cat sample questions, and if satisfied, you can simply proceed to buy one of our subscriptions that suits your need.
Free Trial MCQs
Quiz Summary
0 of 25 Questions completed
Questions:
Information
You have already completed the quiz before. Hence you can not start it again.
Quiz is loading...
You must sign in or sign up to start the quiz.
You must first complete the following:
Results
Results
0 of 25 Questions answered correctly
Your time:
Time has elapsed
You have reached 0 of 0 point(s), (0)
Earned Point(s): 0 of 0, (0)
0 Essay(s) Pending (Possible Point(s): 0)
Average score |
|
Your score |
|
Categories
- Medicine 0%
- Obstetrics & Gynaecology 0%
- Pediatrics 0%
- Psychiatry 0%
- Public Health and Ethics 0%
- Surgery 0%
- 1
- 2
- 3
- 4
- 5
- 6
- 7
- 8
- 9
- 10
- 11
- 12
- 13
- 14
- 15
- 16
- 17
- 18
- 19
- 20
- 21
- 22
- 23
- 24
- 25
- Current
- Review / Skip
- Answered
- Correct
- Incorrect
-
Question 1 of 25
1. Question
1 point(s)Category: PsychiatryA 28-year-old woman complains that her mother is selfish, undereducated, devious, and bereft of any redeeming qualities. On the other hand, she idealizes an aunt whom she describes as kind, sage, and the source of strength in her life. Which of the following defense mechanisms is this woman using?
CorrectIncorrect -
Question 2 of 25
2. Question
1 point(s)Category: PsychiatryA 22-year-old woman with methamphetamine dependence intoxication is brought to the emergency department by the police in a state of extreme agitation. After a brief struggle, she is restrained and sedated. A cursory mental evaluation concludes that she probably has persecutory ideation. Physical examination finds hypertension and tachycardia. She is admitted to a medical psychiatric unit for observation and possible treatment. Which of the following statements about this patient is most accurate?
CorrectIncorrect -
Question 3 of 25
3. Question
1 point(s)Category: PsychiatryA 39-year-old woman consults a psychologist because she feels she needs help in dealing with her 62-year-old mother. She states that ever since her father died last year, her mother constantly nags her about finding a husband and producing grandchildren. The patient states that she has been living with another woman for the past 10 years in a stable, sexual relationship. The couple purchased a home several years ago, have separate and productive careers, travel widely, and do not want to raise children. Which of the following statements about this patient is most likely to be true?
CorrectIncorrect -
Question 4 of 25
4. Question
1 point(s)Category: SurgeryA 65-year-old woman had a 5-year history of pain in the right hip that had gradually gotten worse. She had reached a point where she hobbled with the help of a cane. Nonsteroidal analgesics did not help her very much. She finally agreed to undergo surgery. An x-ray film of the right hip confirmed severe osteoarthritis. There was no osteoporosis. The orthopedic surgeon recommended hip arthroplasty, and she was admitted to the hospital. She was considered at high risk for deep venous thrombosis, and heparin prophylaxis was begun. Which of the following is the best test to monitor heparin therapy?
CorrectIncorrect -
Question 5 of 25
5. Question
1 point(s)Category: SurgeryA 67-year-old man consults his physician because of difficulties while urinating. His major concern is that he has trouble completely emptying his bladder and that after concluding the act, urine sometimes dribbles across the head of his penis, irritating the skin. When asked, he admits that the flow is weaker and slower than it had been and that sometimes it is interrupted. Moreover, he is often awakened during the night by an overpowering urge to urinate. Otherwise, he is healthy. A physical examination was unremarkable, except for findings on a digital rectal examination. This revealed a nontender, smooth but enlarged prostate gland that felt firm but not hard. Routine laboratory workup showed no abnormalities. The prostate-specific antigen (PSA) level was 3.5 ng/dL. Among the following alternatives, which is the best initial treatment to prescribe for this patient?
CorrectIncorrect -
Question 6 of 25
6. Question
1 point(s)Category: PediatricsA 4-day-old, 1,500 g (3lb 5oz) premature infant recovering from a respiratory distress syndrome is noted to have bounding peripheral pulsations and a hyperactive precordium. A continuous machinery murmur is most audible at the left infraclavicular area. Left ventricle hypertrophy (LVH) is present on the electrocardiogram (ECG). The chest x-ray film shows slight enlargement of the heart and increased pulmonary venous markings. Which of the following is the most likely diagnosis?
CorrectIncorrect -
Question 7 of 25
7. Question
1 point(s)Category: MedicineA 58-year old male computer programmer presented to his physician for a routine check-up after his wife nagged him into it. This was his first visit to a physician in at least a decade. After work, he would come home, eat dinner, watch television, and go to bed. He spent his weekends watching sports on television. While doing so, he often ate peanuts, buttered popcorn, or potato chips and downed it with a couple of beers. When asked if this had always been his lifestyle, he replies that prior to turning 40 he played sports and enjoyed dancing with his wife. His subsequent physical examination and laboratory work did not uncover any abnormalities with his heart, lungs, thyroid hormone levels, or kidney function. On the other hand, three different blood pressure readings taken by both the physician and his assistant obtain similar results, namely, an average reading of 178/92 mm Hg. He is 5 feet, 9 inches (1.75 m) tall and weighs 204 pounds (92.5 kg). A fasting blood sugar and lipid profile show a glucose level of 124 mg/dL, total cholesterol (TC) level of 234 mg/dL, and low-density lipoprotein (LDL) level of 158 mg/dL. The most appropriate first line of management in this patient would be which of the following?
CorrectIncorrect -
Question 8 of 25
8. Question
1 point(s)Category: MedicineFor approximately the past two decades, a 62-yearold man had been a successful office manager at a new car dealership; for most of this time, he had his first alcoholic drink of the day upon arising and one or two more before arriving at work, then easily finishing off another pint of bourbon after leaving work and before going to bed. He felt alcoholic drinks had little effect on his ability to function and bragged about having a “hollow leg.” Until recently, he had never had anything to drink while at work, and despite his extreme drinking habits, his work had not suffered. Lately, however, he has had an irresistible urge to indulge in a drink or two or more, even during the workday, and he began to exhibit symptoms of impaired performance. Moreover, he had been caught telling untruths to cover up shortcomings in his performance. Consequently, the owner of the dealership threatened to fire him unless he admitted himself into an alcohol and drug rehabilitation center. As part of his admission to such a center, he underwent a thorough physical examination during which an abnormality present in most heavy drinkers one that may be reversed by abstinence was uncovered. This abnormality most likely was which one of the following?
CorrectIncorrect -
Question 9 of 25
9. Question
1 point(s)Category: MedicineA 56-year-old grandmother whose hobby is gardening presents with a red swelling in her right ring finger. She does not remember any specific trauma to the area, but frequently sustains small cuts on her hands while tending to her garden. Physical examination reveals a small, red, nontender papule on the right ring finger, which shows erythema and swelling, with a small draining pustule on the lateral side. There are ascending erythematous streaks up the right arm, with several draining pustules along their course. Bacterial cultures are negative. Which one of the following is the most effective medication, and most likely to be well-tolerated by this patient?
CorrectIncorrect -
Question 10 of 25
10. Question
1 point(s)Category: MedicineA 40-year-old nurse has been complaining for years about long work hours, poor pay, and lack of concern from her supervisors for her well-being. However, despite her constant complaining, she reports to work punctually and gets her work done in a yeoman-like fashion. She starts to also complain of recurrent episodes of forgetfulness, associated with bouts of sweating, palpitations, anxiety, tremulousness, and fainting. As a consequence, she asks for a medical leave with pay. Before granting her request, she is given a physical examination. Laboratory studies show a serum glucose level of 55 mg/dL (normal, 70-110 mg/ dL), elevated serum insulin levels, and a suppressed level of serum C-peptide. Which of the following is the most likely cause of the hypoglycemia?
CorrectIncorrect -
Question 11 of 25
11. Question
1 point(s)Category: MedicineA 30 year old woman complains of fatigue and dyspnea for the past 2 months. She reports that she has also lost 15 pounds during this time. She has been previously healthy and is not taking any medications. She is pale and thin and has a flow murmur on her cardiac examination. She also has mildly enlarged supraclavicular lymph nodes. Laboratory results are notable for a hematocrit of 30%, mean corpuscular volume (MCV) of 78 µm3, decreased transferrin iron binding capacity (TIBC), and increased ferritin. A screening erythrocyte protoporphyrin level is <35 µg/dL, and a blood smear shows microcytic red cells. Which of the following is the most likely diagnosis?
CorrectIncorrect -
Question 12 of 25
12. Question
1 point(s)Category: MedicineAn otherwise healthy 15 year old boy undergoes evaluation for a newly diagnosed ventricular septal defect (VSD). The boy is at the 50th percentile for height and 45th percentile for weight. He plays soccer regularly with his school team and has never had any significant health problems. His blood pressure is 120/76 mm Hg, pulse is 72/min, and respirations are 11/min. An ECG is unremarkable, but echocardiography demonstrates a defect in the upper interventricular septum with cardiac chambers of normal size. Doppler ultrasound and radionuclide flow studies reveal a small left-to-right shunt with a pulmonary-to-systemic flow ratio of less than 1.5. Which of the following is the most significant complication of this patient's condition?
CorrectIncorrect -
Question 13 of 25
13. Question
1 point(s)Category: MedicineA 54 year old man comes to the physician because of right leg pain for 4 months. He describes it as a deep pain in the calf muscles that occurs intermittently after walking for a given distance and subsides after a few minutes of rest. He has been smoking 20-30 cigarettes a day for the past 25 years and drinks alcohol occasionally. His temperature is 36.8 C (98.2 F), blood pressure is 154/93 mm Hg, pulse is 74/min, and respirations are 13/min. Examination shows thinning of the right calf compared with the left, as well as hair loss on the right leg. Reflexes and sensation are normal. Femoral pulses are normal on both sides, but the right popliteal and pedal pulses are barely detectable. Which of the following is the most likely diagnosis?
CorrectIncorrect -
Question 14 of 25
14. Question
1 point(s)Category: MedicineA 30 year old man presents with a rapidly enlarging, single, stony hard, palpable 2.5-cm nodule in his thyroid gland. Thyroid isotope scanning demonstrates the nodule to be "cold."On resection of the thyroid gland with subsequent pathologic examination, the nodule is found to contain follicular structures, some of which have inwardly protruding fibrovascular branching cores covered by epithelial cells. Many of the epithelial cells have "orphan Annie" nuclei. Which of the following is the most likely diagnosis?
CorrectIncorrect -
Question 15 of 25
15. Question
1 point(s)Category: PediatricsA 2-year-old boy who emigrated from Eastern Europe l year ago is brought to the physician because of fever, cough, and night sweats for 3 weeks. The child's grandmother, who lives with him, has similar symptoms. The child's temperature is 39.2 C (102.6 F), blood pressure is 110/65 mm Hg, pulse is 90/min, and respirations are 28/min. A Mantoux test is reactive, and a chest x-ray film shows a right middle lobe infiltrate and hilar lymphadenopathy. Which of the following is the most appropriate next step in diagnosis?
CorrectIncorrect -
Question 16 of 25
16. Question
1 point(s)Category: PediatricsA 7-year-old girl was found in a routine health supervision visit to have bilateral breast tissue development. She also had long, pigmented hair over the labia majora. Her height and weight are both at the 80th percentile for her age. Which of the following is the most appropriate management?
CorrectIncorrect -
Question 17 of 25
17. Question
1 point(s)Category: Obstetrics & GynaecologyA 17-year-old woman presents to the emergency department with sudden onset of abdominal pain. She was playing in the school band when it began. She describes the pain as starting 30 minutes ago in the left lower quadrant without any radiation. The pain is 7 on a scale of 1 to 10 and not associated with nausea, vomiting, or diarrhea. Onset of menarche was age 13. Although initially her menses were irregular, she has had regular menstrual periods for the past 6 months. She has no fever, and her vital signs are stable. She is sexually active and is on combination oral contraceptive pills. She denies taking other medications. Physical examination reveals a flat abdomen with normal peristalsis. Pelvic examination reveals a normal vagina with a normal-appearing cervix. There is no mucopurulent cervical discharge. Bimanual examination is remarkable with a tender 5-cm mass in the left adnexa. A pregnancy test result is negative. A pelvic sonogram exhibits a normal intrauterine pregnancy and a 5 X 6 cm complex mass of the left ovary, with focal areas of calcification. Which of the following is the most likely diagnosis?
CorrectIncorrect -
Question 18 of 25
18. Question
1 point(s)Category: Obstetrics & GynaecologyA 19-year-old woman presents to the physician's office for routine physical examination and Pap smear. She has no complaints. She has had 2 sexual partners in the past six months and takes oral contraceptive pills. She has no significant past medical history and takes no other medications. She has no known drug allergies. Her temperature is 37.2 °C (98.9 °F) and blood pressure is 120/72 mm Hg. Complete physical exam including pelvic examination is unremarkable. Cervical swab is sent for nucleic acid amplification of Chlamydia trachomatis and Neisseria gonorrhoeae. One week later, the nucleic acid amplification test returns positive for Chlamydia infection. The patient is still asymptomatic. What is the most appropriate next step in the management?
CorrectIncorrect -
Question 19 of 25
19. Question
1 point(s)Category: Obstetrics & GynaecologyA 25-year-old G 1 PO woman at 39 weeks gestation by last menstrual period confirmed by first trimester ultrasound presents to the hospital with complaints of vulvar pain and a "bump" on her vulva. On examination you see clear vesicles and inguinal adenopathy. No cervical or vaginal lesions are present. She is 2 cm dilated, 50 % effaced and at -2 station. Feta I heart rate and contraction monitoring is started. She is contracting regularly. No abnormalities are seen. Which of the following is the most effective intervention to reduce neonatal morbidity in this patient?
CorrectIncorrect -
Question 20 of 25
20. Question
1 point(s)Category: SurgeryA 19-year-old gang member is shot in the abdomen with a .38 caliber revolver. The entry wound is in the epigastrium, to the left of the midline. The bullet is lodged in the psoas muscle on the right. He is hemodynamically stable, and the abdomen is moderately tender. Which of the following is the most appropriate next step in diagnosis?
CorrectIncorrect -
Question 21 of 25
21. Question
1 point(s)Category: SurgeryA young man is shot in the upper part of the neck with a .22 caliber revolver. Inspection of the entrance and exit wounds indicates that the trajectory of the bullet is all above the level of the angle of the mandible, but below the skull. He is fully conscious and neurologically intact. A steady trickle of blood flows from both wounds, and it does not seem to respond to local pressure. He is hemodynamically stable. Which of the following is the most appropriate next step in diagnosis?
CorrectIncorrect -
Question 22 of 25
22. Question
1 point(s)Category: SurgeryA 45-year-old man comes to the emergency department because of severe right flank pain that began abruptly 3 hours ago. The pain comes in waves and radiates down to the ipsilateral testis. The patient is nauseated and extremely restless. His temperature is 37.0 C (98.6 F). Dipstick examination of urine is positive for hematuria. Urinary pH is 5.8. Which of the following is the most appropriate next step in diagnosis?
CorrectIncorrect -
Question 23 of 25
23. Question
1 point(s)Category: Public Health and EthicsA 62-year-old man is referred to a cardiology clinic at a local teaching hospital that serves as the tertiary care center for the surrounding area. The patient suffered a myocardial infarction 2 months •ago, and has been sent by his primary physician for management of his blood pressure. While reviewing his medical record, the physician finds that the patient's hypertension has been treated only with verapamil. The patient also takes a daily 81-mg aspirin tablet and a statin. His high-density lipoprotein and low-density lipoprotein levels are within the desired range. The cardiologist is surprised that the primary physician had not placed him on a beta-blocker and/or angiotensin-converting enzyme inhibitor for control of his blood pressure given his history of myocardial infarction. Which of the following is the most appropriate statement to make to the patient?
CorrectIncorrect -
Question 24 of 25
24. Question
1 point(s)Category: Public Health and EthicsA 79-year-old man is admitted to the hospital for exacerbation of chronic obstructive pulmonary disease. His other medical problems include coronary artery disease, type 2 diabetes mellitus, and obesity. He has smoked cigarettes daily for more than 50 years. His wife says that she is concerned about his health due to multiple hospitalizations over the last year for various issues relating to his comorbidities. After performing the history and physical examination, the treating physician decides to continue his existing medications for diabetes and heart disease. He is also started on supplemental oxygen, bronchodilators, antibiotics, and systemic corticosteroids for his chronic obstructive pulmonary disease exacerbation. Which of the following topics is the most essential to address during the rest of the admission process?
CorrectIncorrect -
Question 25 of 25
25. Question
1 point(s)Category: Public Health and EthicsA 75-year-old Chinese man is brought to the emergency department by his family with sudden-onset shortness of breath. He speaks only Mandarin Chinese. A medical interview is conducted with the assistance of a hospital interpreter, and the patient reports a chronic cough and a 13.6-kg (30-lb) weight loss over the past 6 months. Chest x-ray demonstrates a large right lung mass and an associated pleural effusion. The patient is hospitalized and undergoes bronchoscopy with biopsy; the pathology report comes back as bronchogenic carcinoma. Before the physician enters the room to discuss the biopsy results, he is approached by the patient's son and other family members. They ask that the physician not disclose any bad news to their father as "it would distress him to know." When the patient is asked about this, he defers to his son's judgment, saying, "I am not interested in knowing." Further discussion indicates that the patient is cognitively intact without signs of mental illness. The son and other family members state that they do not want the patient to receive any further information about his condition, prognosis, or treatment options. Which of the following is the most appropriate response to the patient's son?
CorrectIncorrect